There is exactly 1 pair of parallel sides. 6 3 9

Answers

Answer 1

Answer:

Step-by-step explanation:

The given properties of one pair of parallel sides in the four sided

quadrilateral, indicates that the shape is a trapezium.

Response:

The area of the shape is 35 square unit

Answer 2
Parallel sides=6 and 9Height=3

Area of trapezoid

1/2(Sum of parallel sides)(Height)1/2(6+9)(3)1/2(15(3))12(45)22.5units^2

Related Questions

Solve the equation for y.

2x + 5y = 5


y=25x+1

y=−10x+25

y=−25x+1

y=25x+5

Answers

Answer: C

Step-by-step explanation:

[tex]2x+5y=5[/tex]

[tex]\mathrm{Subtract\:}2x\mathrm{\:from\:both\:sides}[/tex]

[tex]2x+5y-2x=5-2x[/tex]

[tex]\mathrm{Simplify}[/tex]

[tex]5y=5-2x[/tex]

[tex]\mathrm{Divide\:both\:sides\:by\:}5[/tex]

[tex]\frac{5y}{5}=\frac{5}{5}-\frac{2x}{5}[/tex]

[tex]\mathrm{Simplify}[/tex]

[tex]\bf{y=-\frac{2}{5}x + 1}[/tex]

w = 4, x = 5, and y = 6
/ = fraction

8x - (6 + y)
w 3/4 = x²
y² - 0.5w
w/4 + 3(x+y)

Answers

Answer:

28, 28, 34, 34

Step-by-step explanation:

8x - (6 + y)

8(5) - (6 + 6)

40-12

28

w ¾ + x²

4(¾) + 5²

3 + 25

28

y² - 0.5w

6² - 0.5(4)

36 - 2

34

w / 4 + 3(x + y)

4 / 4 + 3(5 + 6)

1 + 3(11)

34

An airplane flying with the wind takes 5 hours to travel a distance of 960 miles. The return trip takes 6 hours flying against the wind. What is the speed of the airplane in still air and how fast is the wind blowing? ​

Answers

let's recall that d = rt, distance = rate * time.

a = speed of the airplane in still air.

w = speed of the wind

the distance travelled with the wind is 960 miles, so the return trip is pretty much just that, 960 miles.

when the airplane is going with with wind, the airplane is really not going as fast as "a" mph, is really going faster due to the wind, so the speed of the plane is "a + w", because the wind is adding its speed to it.

when the airplane is going against the wind is not really going "a" mph fast either, is really going slower at "a - w" mph, because the wind is subtracting speed from it, so

[tex]\begin{array}{lcccl} &\stackrel{miles}{distance}&\stackrel{mph}{rate}&\stackrel{hours}{time}\\ \cline{2-4}&\\ \textit{with the wind}&960&a+w&5\\ \textit{against the wind}&960&a-w&6 \end{array}~\hfill \begin{cases} 960=(a+w)(5)\\\\ 960=(a-w)(6) \end{cases} \\\\\\ \stackrel{\textit{using the 1st equation}}{960=(a+w)(5)}\implies \cfrac{960}{5}= a + w\implies 192=a+w\implies \boxed{192-w=a}[/tex]

[tex]\stackrel{\textit{substituting on the 2nd equation}}{960=(\underline{192-w}~~ - ~~w)(6)}\implies \cfrac{960}{6}=192-w-w\implies 160=192-2w \\\\\\ 2w+160=192\implies 2w=32\implies w=\cfrac{32}{2}\implies \blacktriangleright w=16 \blacktriangleleft ~\hfill \blacktriangleright \stackrel{192 - 16}{176=a} \blacktriangleleft[/tex]

Select the correct answer
What is the range of this function?
Please help it’s multiple choice wil choose brain list an 100 points!!!!!

Answers

LETTER B IS THE CORRECT ANSWER

Subtract. Write your answer in simplest for. 2/3 - 3/8

Answers

The lowest common denominator of 3 and 8 is 24, thus you must multiply 2/3 by 8/8 and 3/8 by 3/3. This is 16/24-9/24, which is equal to 7/24.
First we need common denominators so we can multiply 2/3 by 8 and 3/8 by 3. Now we would have 16/24 - 9/24 which would be 7/24. 7/24 cannot be simplified because 7 cannot be multiplied by anything to get 24. The answer would be 7/24

5-3x5* Whats the answer

Answers

Answer:

-10

Step-by-step explanation:

5 - 3*5

= 5 - 15

= -10

Sunshine Rental rents a motorcycle at a daily rate of $57.99 plus 48 cents per mile. City Rentals
rents the same motorcycle at $58.95 plus 46 cents per mile. For what mileage is the cost the same?

Answers

Answer:

48 miles

Step-by-step explanation:

To find the mileage when both cost the same, first form two equations for the total cost for each rental place.

Let the mileage be m.

Sunshine Rental

Daily rate= $57.99

Total cost= $(57.99 +0.48m)

City Rentals

Daily rate= $58.95

Total cost= $(58.95 +0.46m)

Let's equate both equations together.

When both costs the same,

57.99 +0.48m= 58.95 +0.46m

0.48m -0.46m= 58.95 -57.99

0.02m= 0.96

Solve for m:

m= 0.96 ÷0.02

m= 48

The cost is the same at 48 miles.

Find the average of this set of numbers.28, 26, 21, 13
A) 22
B) 20
C) 19
D) none of the above

Answers

Answer:

A

Step-by-step explanation:

26 + 28 = 54

54 + 21 = 75

75 + 13 = 88

88 / 4 = 22

Answer is A

Answer: 22

Step-by-step explanation: Add all of the numbers in the data set: 28+26+21+13= 88

Then divide 88 by 4: 22

What is M G to the nearest tenth

Answers

Answer:

tan m‹G = 28.2/45.8

m‹G= 31.62°

Determine f”(1) for the function f(x)=(3x^2-5x^2)^2.

Answers

Answer: 48

Step-by-step explanation:

Flying against the jetstream, a jet travels 2370 miles in 3 hours. Flying with the jetstream, the same jet travels 6350 miles in 5 hours. What is the rate of the jet in still air and what is the rate of the jetstream? ​

Answers

Velocity of jet in still air is

970

miles per hour and velocity of wind is

210

miles per hour.

Explanation:

Jet's velocity against wind is

3040

4

=

760

mile per hour and flying with wind it is

8260

7

=

1180

miles per hour.

Let the velocity of jet in still air be

x

miles per hour and velocity of wind be

y

miles per hour.

As such its velocity against wind is

x

y

and with wind is

x

+

y

and therefore

x

y

=

760

and

x

+

y

=

1180

Adding the two

2

x

=

1940

and

x

=

970

and

y

=

1180

970

=

210

Hence velocity of jet in still air is

970

miles per hour and velocity of wind is

210

miles per hour.

The speed of an aircraft in still air is 200kmh−1. The wind blows from the west at a speed of 85.0kmh−1.

1 Expert Answer

The speed of the plane in still air is 110 mph.

Multiply the headwind or tailwind component by your estimated flight time expressed as a decimal, such as "3.2 hours." Add the result (don't forget that tailwinds are negative numbers) to the geographic distance of your route. The resulting number will be total nautical air miles of your trip.

f(x) = -x^2 + 6x - 10
Sketch the graph of each function.

(Please show work, I don’t understand).

Answers

Answer:

The graph would not be as able to see the form and/or function so hope this helps

How many units are there between point A (-3,80) and point B (-3,12)

Answers

Answer:

68 units

Step-by-step explanation:

Given:

Point A = (-3, 80)Point B = (-3, 12)

As the x-values of points A and B are the same (x = -3), the line through points A and B is vertical.

Therefore, to find the distance between the two points, simply subtract the y-value of point B from the y-value of point A:

[tex]\sf distance=y_A-y_B=80-12=68 \ units[/tex]


Please answer question in screen shot

Answers

Answer:

1 (the answer will always be 1)

Step-by-step explanation:

Let's forget about the absolute value signs for a second and just look at what's inside

[tex]\frac{x+y}{-x-y}[/tex]

This can be rewritten as

[tex]-\frac{x+y}{x+y}[/tex]

When you have the same numerator and denominator, it will always come out to 1, so we can just say that this is saying

[tex]-\frac{x+y}{x+y} = -1[/tex]

Now, back to the absolute value signs

This comes out to negative 1, but the absolute value signs means to always change the negative to a positive (but if it's a positive, it stays positive!)

So that means:

[tex]|-1| = 1[/tex]

and 1 is your answer

A string of decorative lights has 32 bulbs for every 2 feet of wire.
What is the unit rate of bulbs per yard?


HELP !!!!!

Answers

Answer:

First convert two feet into yards, knowing that:

1 foot = 0.333... yards

So,

2 feet in yards would mean double of 0.333... because one single foot is 0.333... yards.

2(0.333...)

= 0.666..., which we can round to 0.67.

So our new ratio will be 32 bulbs per 0.67 yards.

Now we convert this ratio into a unit rate,

Order your units;

Bulbs         32           47.76

______   ______ = ______, which rounds to about 48.

Yards        0.67             1

So the unit rate is 48 bulbs per yard.

State the following numbers using '+' or '_'.
(a) 80 less than zero
=

(b) 76 more than zero
=


2.List all integers
(a) from -8 to 4
=

(b) from -12 to -2
=​

Answers

Step-by-step explanation:

-80 is less than 0.

+76 is more than 0.

-8, -7, -6, -5, -4, -3, -2, -1, 0, 1, 2, 3, 4

-12, -11, -10, -9, -8, -7, -6, -5, -4, -3, -2

I need help with this​

Answers

Answer:

The correct answer is B: 3 cubic feet.

Step-by-step explanation:

If you want to find the volume of a rectangular prism, you need to know the length, width, and height. The first rectangular prism is a cube with a length of 3 feet, so the volume of the figure would be [tex]3^{3}=27 ft^{3}[/tex]. The second rectangular prism has a length of 3 feet, a width of 2 feet, and a height of 4 feet, so the volume of the prism would be 3 × 2 × 4 = [tex]24 ft^{3}[/tex]. Now that you know the 2 volumes, you need to subtract the volumes in order to find the difference, which is 27 - 24 = 3 [tex]ft^{3}[/tex].

00:23:04
Hide
Product A sells 2.5 times more than Product B. Product B sells 60% less than Product C. Product C sold 25,000 units this month.
How many units of Product A were sold?
10,000
15,000
25,000 37,500 62,500

Answers

Answer:37,500

Step-by-step explanation: Find 60% of 25000 which is 15000 then multiply by 2.5

what is the area of a circle with a circumference of 250

also I lost some of my friends on here and I cant find them and I haven't seen them since June :(

Answers

I hope I can help you:)
btw I still believe that your friends will meet you again in the future, don’t be so sad:))

Which number is bigger: 0.135 or 0.14?

Answers

0.14 as if you convert it into like decimal (add a 0 if the places are not the same e.g. 0.1 and 0.02 here 0.1 does not have the same number of digits so add a zero so it becomes 0.10 :. 0.10 is bigger then 0.02) (note the value of 0.1 will remain the same) so here 0.14 will become 0.140 :. bigger than 0.135

The number 0.14 is bigger than 0.135.

To compare the two numbers, you can look at their digits after the decimal point. In both numbers, the digit in the tenths place is 1. The next digit in 0.14 is 4, which is greater than the corresponding digit in 0.135, which is 3.

Since the digit in the hundredths place (4) is greater than the digit in the hundredths place (3), the number 0.14 is bigger than 0.135.

In summary, the order of the numbers from smallest to largest is:

0.135 < 0.14.

To know more about number:

https://brainly.com/question/3589540

#SPJ2

This is the question

Answers

Check the picture below, so the lines look more or less like so, so the shape that we'll be getting will be the shape of a bowl with a hole in it, thus we'll use the washer method.

the way I use to get the larger Radius is simply using the "area under the curve" method, namely f(x) - g(x), where g(x) in this case is the axis of rotation.

so to get "R" and "r" we can get it by

[tex]\stackrel{y = 1}{(1)}~~ - ~~\stackrel{\stackrel{\textit{axis of rotation}}{y = -3}}{(-3)}\implies 1+3\implies \stackrel{R}{4} \\\\\\ \stackrel{y = \ln(x)}{\ln(x)}~~ - ~~\stackrel{\stackrel{\textit{axis of rotation}}{y = -3}}{(-3)}\implies \stackrel{r}{ln(x)+3} \\\\[-0.35em] ~\dotfill\\\\ \stackrel{R^2}{(4)^2}~~ - ~~\stackrel{r^2}{[\ln(x)+3]^2}\implies 16~~ - ~~[\ln^2(x)+6\ln(x)+9] \\\\\\ \ln^2(x)+6\ln(x)+7~\hfill \boxed{\displaystyle\int~[\ln^2(x)+6\ln(x)+7]dx}[/tex]

Un portapapeles cuesta $2.23. Cuesta $0.58 más que un cuaderno. Lisa compró dos portapapeles
y un cuaderno. Ella pagó con un billete de diez dólares. ¿Cuánto cambio recibió Lisa? Usa un diagrama
de cinta para mostrar tu razonamiento.

Answers

Answer:

3.38

Step-by-step explanation:

primero tienes que restar 2.23 por 0.58 que es igual a 1.65 luego sumas 1.65 por 4.46 porque 2.23 multiplicado por 2 es igual a 4.46 y eso es igual a 6.11 luego tienes que restar 10 por 6.11 que es igual a 3.89

3.-Un camión de pasajeros arranca desde el reposo y mantiene una aceleración de 0.8 mis?
a) ¿En qué tiempo recorrerá una distancia de 0.5 km?
b) ¿Qué rapidez levará en ese tiempo en m/s y en km/h?

Answers

a) El tiempo requerido por el camión para recorrer 0.5 kilómetros es aproximadamente 35.355 segundos.

b) La rapidez que llevará el camión en ese tiempo es 28.284 metros por segundo (101.822 kilómetros por hora).

Caso de movimiento uniformemente acelerado

El camión experimenta un movimiento uniformemente acelerado, que ocurre al considerar que su aceleración ([tex]a[/tex]), en metros por segundo al cuadrado, es constante en el tiempo ([tex]t[/tex]), en segundos.

a) El tiempo requerido es determinado por el siguiente polinomio de segundo orden:

[tex]x = v_{o}\cdot t + \frac{1}{2}\cdot a\cdot t^{2}[/tex]   (1)

Donde:

[tex]x[/tex] - Cambio en la posición, en kilómetros. [tex]v_{o}[/tex] - Rapidez inicial, en metros por segundo.

Si sabemos que [tex]x = 500\,m[/tex], [tex]a = 0.8\,\frac{m}{s}[/tex] y [tex]v_{o} = 0\,\frac{m}{s}[/tex], entonces tenemos que el tiempo requerido es:

[tex]0.4\cdot t^{2}-500 = 0[/tex]

[tex]t \approx 35.355\,s[/tex]

El tiempo requerido por el camión para recorrer 0.5 kilómetros es aproximadamente 35.355 segundos. [tex]\blacksquare[/tex]

b) La rapidez final ([tex]v[/tex]), en metros por segundo, se determina con el siguiente polinomio de primer orden:

[tex]v = v_{o} + a\cdot t[/tex]   (2)

Si sabemos que [tex]v_{o} = 0\,\frac{m}{s}[/tex], [tex]a = 0.8\,\frac{m}{s^{2}}[/tex] y [tex]t = 1250\,s[/tex], entonces tenemos que la rapidez final es:

[tex]v = 0 + 0.8\cdot (35.355)[/tex]

[tex]v = 28.284\,\frac{m}{s}[/tex]   [tex]\left(101.822\,\frac{km}{h} \right)[/tex]

La rapidez que llevará el camión en ese tiempo es 28.284 metros por segundo (101.822 kilómetros por hora). [tex]\blacksquare[/tex]

Para aprender más sobre movimiento uniforme, invitamos a ver esta pregunta verificada: https://brainly.com/question/118814

Which of the following sets of numbers is a Pythagorean triple?

42, 40, 82
42, 40, 58
41, 41, 58
40, 41, 57
HELP QUICK PLEEZ

Answers

The awnser is would be 42,40,58

Answer:

42,40,58

Step-by-step explanation:

The table represents the number of miles to the nearest airport. Find the median.
# OF MILES: 20, 22, 24, 26
FREQUENCY: 3, 2, 1, 4

Answers

The median value in the data set is the average of the two middle values which is: 23.

What is the Median of a Data Set?

In an ordered data set, the median is the middle value or average of the two middle values in the data set.

List out each data point in the data set given:

20, 20, 20, 22, 22, 24, 26, 26, 26, 26

The middle values in the data set are, 22 and 24.

Median = (22 + 24)/2

Median = 23

Learn more about the median on:

https://brainly.com/question/26177250

FIND THE MISSING LENGTH.

Answers

Answer:

  x = 9

  x+7 = 16

Step-by-step explanation:

The triangles in this geometry are similar, so the ratios of corresponding sides are proportional.

  long side/short side = (x+7)/12 = 12/x

  x(x +7) = 12²

  x +7x -144 = 0 . . . . . subtract 144 to put in standard form

  (x +16)(x -9) = 0 . . . . factor

  x = 9 . . . . . . only the positive solution is useful here

The missing lengths are ...

x = DC = 9x+7 = DA = 16

there are 374 students at a school. Each table in the cafeteria hold 16 students. What is the MINIMUM number of tables needed for all of the students to eat lunch at the same time

Answers

Answer:  There should at least be 23 tables

Step-by-step explanation:

If there is 23 tables there will still be six students with nowhere to sit :)

An AlienWare computer cost $4,200. You have a 35% coupon for their products. How much are you able to purchase the computer for?

Answers

Answer:

$1470 coupon of their product I have

Lin got a $50 gift card to an online music store. She uses the gift card to buy an album
for $9.99. She also wants to use the gift card to buy some songs. Each song costs
$1.29.
Which of these inequalities describes this situation, where n is the number of songs Lin
wants to buy?
a) 9.99 + 1.29n > 50
b) 9.99 + 1.29n < 50
c) 9.99 - 1.29n > 50
d) 9.99 - 1.29n < 50

Answers

Answer:

b i think because 50 minus 9.99 then 1.29 will be less than cuz it cant go over

These numbers are common multiples of ________.
15, 30, 45, 60

A) 2 and 3
B) 3 and 5
C) 4 and 5
D) 3 and 4

Answers

B is the answerrRe.
.

Answer:

The answer is B.

Step-by-step explanation:

15÷3=5    15÷5=3

30÷3=10   30÷5=6

45÷3=15    45÷5=9

60÷3=20    60÷5=12

Other Questions
I need your help!!Ex 5 The lettet ___this morning Say a few words about the energy problems in your country.1. Which of the kinds of energy are used in your country now? 2. Are nuclear power stations the only way out for Armenia?3. Which kind of energy do you think is the best for your country? worst? most practical? least practical? Urgent!!! Need help Please!!Find the area of the triangle. a. 620 sq. m b. 270 sq. m c. 540 sq. m d. 980 sq. m points to make your day better :D What length is X 3.00004.0000 The image shows an aspect of human culture. Which aspect is illustrated in the image? How much does it cost to hike the appalachian trail?. Committed time includes time needed for all of the following EXCEPT:A. maintaining a homeB. raising a familyC. playing sportsD. attending schoolPlease select the best answer from the choices provided. Fear of being stigmatized often discourages individuals from seeking treatment for their mental illness. Please select the best answer from the choices provided T F. Long answer questions. 1. Explain the trend of literacy in Nepal based on the statistics given in the text. Where are the most greenhouse gases generated during the production of electricity? Explain one way in which the actions of the chilean military as described in the passage reflect the global political context of the late twentieth century Which graph shows the line of best fit for the data ? Please help me with this equation, add process too. Thanks A perfectly competitive firm has fixed costs of $1 million a month, variable costs of $2 million a month and product sales of $3 million a month. what statement is a correct analysis of the situation 8-1 skills practice multiplying and dividing rational expressions Five widgets and three gadgets cost $109. 90. One widget and four gadgets cost $75. 70. How much does one gadget cost? how does a child become socialized by the family? *HELP ASAP I WILL GIVE U BRAINLEAST* (20 POINTS)You have inherited $8000 from your grandfather. Knowing that you will soon be going to college you decide to invest your money into an account that has an 8% interest rate that is compounded quarterly. How much money will you have in your account after 5 years?a) Exponential growth or decay:b) Identify the initial amount:c) Identify the growth/decay factor:d) Write an exponential function to model the situation:e) Do the problem: